展开全部

题目列表

题目内容
The average weight of 7 fish is 25 pounds. Among them, the 4 heaviest weigh 117 pounds in total, the average of the 4 lightest weigh 20.5 pounds. What`s the median weight of the 7 fish? pounds
x < y, and the median of x, y and 120 is 100

Quantity A

x

Quantity B

90


All the angles in a regular n-sided polygon and a (n+1)-sided regular polygon are included in a list.

Quantity A

The median of the list of angles

Quantity B

90°


The median of 5 integers is 60, with each of the remaining 4 numbers between 50 and 70, inclusive. Which of the following could be the sum of the 5 integers?
There are 15 numbers in List M, in which 8 numbers are greater than 50.4, and 8 numbers are less than 54.6.

Quantity A

The median of the numbers in List M

Quantity B

52.5


a < b < c < d

Quantity A

$$\frac{a+b}{2}$$

Quantity B

The median of four numbers


If the median of seven consecutive even integers is 2n+2, then what is the average of them?
In a certain sequence of 90, 105, x, 120 and 125, both the mean and the median are equal to x. What is the value of x?
The average of 10 positive integers is 7.5.

Quantity A

The probability of randomly selecting a number that is greater than 7

Quantity B

0.5


The range of 7 integers is 4, and the median of the 7 integers is m. The greatest number less than the median is m-3.

Quantity A

The mean of the 7 integers

Quantity B

m


The stock prices of three companies are x, 2x and x+2, respectively and x>2. Which of the following statement alone can be sufficient to determine the value of x?
Indicate all such statements.
List R and List S both have 100 numbers, among which 50 are overlapping. The mean of List R is smaller than the mean of List S.

Quantity A

The median of List R

Quantity B

The median of List S


7, 11, 15, 19, 23, x, y The average of the above 7 positive integers is 13. Which of the following could be the median of the 7 numbers?
Indicate all such numbers.
Set A consists of 5 numbers. The median of the 5 numbers is m, the difference between m and the maximum number is 6, while the difference between m and the minimum is 2.

Quantity A

m

Quantity B

The average of the 5 numbers


Set A={6, 8, 12, 14 and x} If x increases from 12 to 14, then which of the following values will also change?
Indicate all such values.
The median of 7 positive integers is 10, and the average of these 7 numbers is 9. The only mode is 2. What is the greatest possible number in the set?

Quantity A

The range of seven consecutive integers

Quantity B

7


A list contains 10 different integers.

Quantity A

The range of the 10 integers

Quantity B

10


Set S includes all the two-digit integers that are multiples of 3 or 5. What is the range of the set of numbers?
The range of a list is 18. If the smallest number is -3, what is the value of the greatest number?

共收录:

25000 +道题目

202本备考书籍

最新提问